Сила тока через напряжение: 1. Электрическое сопротивление. Закон Ома для участка электрической цепи

Содержание

1. Какая связь существует между напряжением, током и сопротивлением? | 2. Закон Ома | Часть1

1. Какая связь существует между напряжением, током и сопротивлением?

Какая связь существует между напряжением, током и сопротивлением?

Электрическая цепь считается сформированной тогда, когда создан такой проводящий путь, который позволяет свободным электронам непрерывно перемещаться. Это непрерывное движение свободных электронов по проводникам цепи называется током. Иногда его, по аналогии с потоком воды через трубу, называют «потоком».

Сила побуждающая электроны «течь» по цепи называется напряжением. Напряжение — это определённая мера потенциальной энергии, которая всегда взаимосвязана с двумя точками цепи. Когда мы говорим что в схеме присутствует определенная величина напряжения, мы имеем в виду величину потенциальной энергии

, необходимой для перемещения электронов из одной точки цепи в другую. Без привязки к двум конкретным точкам цепи термин «напряжение» не имеет смысла.

При движении свободных электронов через проводники, определенное воздействие на них оказывает сила трения, которая препятствует движению. Это противодействие движению называется сопротивлением. Величина тока в цепи зависит от величины напряжения, заставляющего электроны двигаться, а так же от величины сопротивления, тормозящего поток электронов. Так же как и напряжение, сопротивление взаимосвязано с двумя точками цепи.

Чтобы конкретизировать понятия величины тока, напряжения и сопротивления, мы должны присвоить им единицы измерения, точно также, как единицы измерения присвоены массе, температуре, объему, длине и другим видам физических величин. Например, для массы мы используем единицу измерения «килограмм» или «грамм», для температуры — градус Фаренгейта или градус Цельсия. Стандартные единицы измерения силы тока, напряжения и сопротивления приведены в таблице:

«Обозначение» каждой величины — это буква латинского алфавита, которая используется для представления величины в алгебраическом уравнении.  Использование латинских букв в физических и технических дисциплинах признано на международном уровне. «Аббревиатура» представляет собой первую букву единицы измерения на русском и английском языках. Исключение составляет аббревиатура слова Ом, которую в английской версии представляет буква греческого алфавита.

Каждая единица измерения названа в честь известного экспериментатора в области электроники: Ампер — в честь француза Ампера Андре Мари,

Вольт — в честь итальянца Алессандро Вольта, Ом — в честь немца Ома Георга Симона.

Обозначение каждой величины имеет определенный смысл. Буква «R» (resistance) для сопротивления говорит сама за себя. Напряжение в нашей стране обозначается буквой «U», а за границей оно обозначается буквой «V» (voltage), что тоже говори само за себя. Что касается буквы «I» для обозначения силы тока, и буквы «E» — для второго обозначения напряжения, то они немного не вписываются в это правило.  «I», как многие полагают, означает  «Intensity» (Интенсивность (потока электронов)), а «E» — «Electromotive force» (Электродвижущую силу). Обозначения «E» и «U» по большей части являются взаимозаменяемыми, однако, некоторые радиолюбители резервируют букву «E» для обозначения напряжения источника питания (батареи, генератора и др.), а буквой «U» обозначают напряжение чего-нибудь еще.

Все эти обозначения используют заглавные буквы, кроме случаев, когда величина (особенно напряжения или тока) описывается в пределах короткого промежутка времени (так называемое «мгновенное» значение). Например, стабильное на протяжении длительного периода времени напряжение батареи обозначается заглавной буквой «E», а пиковое напряжение в момент удара молнии в линию электропередач скорее всего  будет обозначено строчной буквой «e» (или «u»). Это же правило применяется и к силе тока, где строчная буква «i» обозначает силу тока в определенный момент времени. Большинство измерений постоянного тока (DC) обозначается заглавными буквами, потому что он стабилен с течением времени.

Одной из основополагающих, но редко используемых единиц измерения в электронике является кулон. Кулон это мера электрического заряда, он пропорционален количеству свободных электронов. Один кулон равен 6,250,000,000,000,000,000 электронов. Величина электрического заряда обозначается буквой «Q», а аббревиатура кулона — буква «C» (coulomb). 1 Амер (единица измерения потока электронов) равен 1 Кулону электронов, проходящих через определенную точку цепи за 1 секунду времени. Иными словами, электрический

ток — это скорость движения электрического заряда через проводник.

Как было сказано выше, напряжение — это количество потенциальной энергии на единицу электрического заряда, необходимой для перемещения электронов из одной точки цепи  в другую. Поэтому, прежде чем мы сможем точно определить что из себя представляет «Вольт», мы должны понять, как измерить величину называемую «потенциальной энергией». Общей единицей измерения для любой энергии является

джоуль. Джоуль равен работе, совершаемой при перемещении точки приложения силы, равной одному ньютону, на расстояние одного метра в направлении действия силы. Исходя из вышеприведенного определения напряжения, 1 Вольт равен 1 Джоулю электрической потенциальной энергии на (деленному на) 1 Кулон заряда. Таким образом, 9-вольтовая батарея затрачивает 9 джоулей энергии на перемещение каждого кулона электронов через цепь.

Рассмотренные обозначения и единицы измерения электрических величин очень важно знать, так как мы, с настоящего момента, начинаем исследовать соотношения между ними в электрических цепях. Первым, и возможно самым важным соотношением между током, напряжением и сопротивлением является закон Ома, открытый и опубликованный Георгом Симоном Омом в 1827 году. Основным открытием Ома было то, что сила тока в проводнике прямопропорциональна напряжению, приложенному к его концам.

Ом выразил своё открытие в виде простого уравнения, описывающего взаимосвязь тока, напряжения и сопротивления:

В этом алгебраическом выражении сила тока (I) прямопропорциональна напряжению (U) и обратно пропорциональна сопротивлению (R). Используя формулу закона Ома и методы алгебры, можно вычислить напряжение и сопротивление:

Давайте посмотрим, как эти уравнения работают при анализе простых электрических схем:

В приведенной выше схеме есть только один источник напряжения (батарея слева) и одно сопротивление току (лампа справа). Это делает ее очень простой в применении закона Ома. Если нам известны значения любых двух из трех величин (силы тока, напряжения и сопротивления) в этой схеме, то используя закон Ома, мы можем вычислить третью.

В первом примере мы вычислим силу тока (I) при заданных значениях напряжения (U) и сопротивления (R):

Чему равна сила тока (I) в этой схеме?

Во втором примере мы вычислим сопротивление (R) при заданных значениях напряжения (U) и силы тока (I):

Чему равно сопротивление (R) лампы?

В последнем примере мы вычислим величину поставляемого батареей напряжения (U) при заданных значениях силы тока (I) и сопротивления (R):

Чему равно поставляемое батареей напряжение (U)?

Закон Ома очень простой и полезный инструмент для анализа электрических цепей.

Он так часто используется при обучении электронике, что намертво врезается в память серьезных студентов. Для тех-же, кто не дружит с алгеброй, существует небольшая уловка для запоминания этого закона. Единственное что нужно сделать, это заключить буквы U, I  и R  в треугольник следующим образом:

Если вам известны значения U и I, и нужно вычислить R, то просто зачеркните эту букву в треугольнике, и вы увидите что нужно сделать:

Аналогичным образом можно вычислить значения I и U:

Краткий обзор:

  • Напряжение измеряется в вольтах, и обозначается буквами «E» или «U».

  • Сила тока измеряется в амперах, и обозначается буквой «I».

  • Сопротивление измеряется в омах, и обозначается буквой «R».

  • Закон Ома: I = E/R ; U = IR ; R = E/I.

Зависимость мощности от силы тока, формула мощности, физический смысл

Первое упоминание об электричестве встречается в опытах древнегреческого философа Фалеса. Именно он первым обнаружил, что предметы при трении притягиваются. Одноименный термин был введен в начале 17-го века английским физиком Гилбертом, после опытов, проведенных с магнитами. Отцом же науки об электричестве считается французский ученый Кулон – именно после открытия закона, получившего его имя, электротехника начала свою победную поступь, которая продолжается до сих пор. Этот закон утверждает, что два точечных заряда в безвоздушной среде взаимодействуют с силой, прямо пропорциональной их модулям и обратно – расстоянию между ними, возведенному в квадрат.

Выясним, что же представляет собой понятие электричество?

Если коротко, то это – направленное движение потока заряженных частиц. Тела, через которые они проходят, называются проводниками. Каждый проводник имеет определенное сопротивление электрическому току, которое раз

И, перед тем, как перейти к основным законам, несколько слов о заряженных частицах: они бывают, условно говоря, положительными и отрицательными. Одноименные заряды отталкиваются, а разноименные – притягиваются.

А теперь, перейдем к главному.

Основа-основ науки об электричестве – закон Ома.

Эксперимент, который провел этот немецкий физик, привел его к следующему убеждению: сила тока I, проходящего через металлический проводник, пропорциональна напряжению на его концах, или I = U/R

Здесь напряжением называется разность, образно говоря, «давлений», созданных двумя точками электрической цепи. Измеряют его в вольтах. Электрический ток представляет собой число электронов, которые пропускает участок электрической цепи и измеряется в амперах. Сопротивлением считается свойство цепи помешать этому движению. В честь упомянутого физика, его измеряют в омах. Иначе говоря, проводник, через который проходит ток в 1 ампер при напряжении в 1 вольт, обладает сопротивлением в 1 ом.

Вся остальная электротехника «пляшет» от этого.

О мощности электрического тока

В физике мощностью считают скорость выполнения работы. Неважно, какой. Чем эта операция проводится быстрее, тем большей считается мощность того, кто ее исполняет, будь то человек, механическое устройство или что-то еще.

Так же и в случае с электрическим током: ее мощность представляет собой отношение работы, произведенной движущимися электрическими зарядами к промежутку времени, которое для этого понадобилось.

Проще говоря, для того, чтобы получить электрическую мощность в 1 ватт, когда источник тока имеет напряжение 1 вольт, необходимо пропустить через проводник ток в 1 ампер. Другими словами, мощность (P) можно посчитать, перемножив друг на друга электрическое напряжение и ток:

P = U*I.

Запомнив эту нехитрую формулу, на практике можно рассчитать мощность. Например, если известны значения тока и сопротивления, а о напряжении сведений нет, можем воспользоваться законом Ома, подставив в формулу вместо него I*R. Получится, что мощность равна квадрату электрического тока, помноженному на сопротивление.

Этот закон точно так же придет на помощь, если известны величины напряжения и сопротивления. В этом случае подставив вместо значения тока I = U/R, получим значение мощности, равное квадрату напряжения, поделенному на сопротивление.

Вот так – ничего сложного!

Конспект урока по физике на тему «Связь силы тока и напряжения. Закон Ома для участка цепи»

Конспект урока по физике по теме

«Связь силы тока и напряжения. Закон Ома для участка цепи»

8 класс

Цель обучения: изучение зависимости силы тока в участке цепи (проводнике) от напряжения на этом участке и его сопротивления; усвоение аналитического выражения закона Ома для участка цепи; формирование умений строить и читать графики зависимости силы тока от напряжения при постоянном сопротивлении и силы тока от сопротивления при постоянном напряжении.

Ход урока

  1. Актуализация знаний (проводится в ходе опроса учащихся)

  1. Как включается в цепь вольтметр?

  2. Как включается в цепь амперметр?

  1. В какой схеме (1-4) неправильно включен амперметр? ____

  2. В какой схеме (1-4) неправильно включен вольтметр?____

  3. В какой схеме (1-4) выключатель не играет роли?____

  1. Постановка проблемного вопроса

Почему сила тока в сварочном аппарате в десятки миллионов раз больше, чем в электронных часах?

  1. Изучение нового материала

Учащиеся вместе с учителем разбирают опыты по теме «Связь силы тока и напряжения. Закон Ома для участка цепи», по результату которых делают запись в тетрадях, выводы.

  1. Сила тока в проводнике прямо пропорциональна приложенному напряжению и обратно пропорциональна сопротивлению проводника.

  2. I=, где I- сила тока (А), U- напряжение (В), R- сопротивление (Ом).

  3. Составление «треугольника памяти».

  4. Чем меньше сопротивление вольтметра, тем меньше изменения вызывают они при подключении в цепь.

  5. Вольт-амперная характеристика проводника представляет собой график зависимости силы тока от напряжения.

  1. Использование цепей без нагрузки приводит к недопустимо опасному росту силы тока (короткому замыканию).

  1. Физкультминутка

  2. Решение задач

  1. Из каких элементов состоит цепь на рисунку? Будет ли идти ток через сопротивление R, если ключи 1 и 2 разомкнуть? Будет ли идти ток и через какие элементы цепи, если замкнуть: а)только ключ 1; б) только ключ 2; в) оба ключа?

  1. Заполните таблицу:

Постройте график зависимости тока от напряжения.
  1. Какое сопротивление имеет вольтметр, рассчитанный на 127В, если по нему течет ток, равный 0,02А?

  2. В трамвайной сети напряжение 575В. Средняя сила тока, проходящего по обмотке трамвайного мотора 71А. Каково сопротивление обмотки?

  3. Вольтметр показывает напряжение 2,5В на концах участка цепи сопротивлением 1,4Ом. Включенный в эту же цепь амперметр показывает силу тока 1,8А. Верны ли показания амперметра?

  1. Закрепление знаний

  1. Связь между какими физическими величинами устанавливает закон Ома для участка цепи?

  2. Может ли в проводнике протекать ток: а) очень большой силы при малом напряжении; б) малой силы при большом напряжении?

  1. Рефлексия

Бистро ли я усваиваю определения и формулы? Сколько раз я повторяю то, что нужно хорошо знать?

  1. Дом. задание

  2. Результат урока

Учащиеся узнают формулировку и смысл закона Ома для участка цепи, формулу, выражающую закон; научатся находить значения силы тока, напряжение и сопротивление, связанные формулой закона Ома.

Корта урока (для учащихся)

  1. Актуализация знаний (проводится в ходе опроса учащихся)

  1. Как включается в цепь вольтметр?

  2. Как включается в цепь амперметр?

  1. В какой схеме (1-4) неправильно включен амперметр? ____

  2. В какой схеме (1-4) неправильно включен вольтметр?____

  3. В какой схеме (1-4) выключатель не играет роли?____

  1. Проблемный вопрос

Почему сила тока в сварочном аппарате в десятки миллионов раз больше, чем в электронных часах?

  1. Изучение нового материала

  1. Сила тока в проводнике прямо пропорциональна приложенному напряжению и обратно пропорциональна сопротивлению проводника.

  2. I=, где I- сила тока (А), U- напряжение (В), R- сопротивление (Ом).

  3. Составление «треугольника памяти».

  4. Чем меньше сопротивление вольтметра, тем меньше изменения вызывают они при подключении в цепь.

  5. Вольт-амперная характеристика проводника представляет собой график зависимости силы тока от напряжения.

  1. Использование цепей без нагрузки приводит к недопустимо опасному росту силы тока (короткому замыканию).

  1. Решение задач

  1. Из каких элементов состоит цепь на рисунку? Будет ли идти ток через сопротивление R, если ключи 1 и 2 разомкнуть? Будет ли идти ток и через какие элементы цепи, если замкнуть: а)только ключ 1; б) только ключ 2; в) оба ключа?

  1. Заполните таблицу:

Постройте график зависимости тока от напряжения.
  1. Какое сопротивление имеет вольтметр, рассчитанный на 127В, если по нему течет ток, равный 0,02А?

  2. В трамвайной сети напряжение 575В. Средняя сила тока, проходящего по обмотке трамвайного мотора 71А. Каково сопротивление обмотки?

  3. Вольтметр показывает напряжение 2,5В на концах участка цепи сопротивлением 1,4Ом. Включенный в эту же цепь амперметр показывает силу тока 1,8А. Верны ли показания амперметра?

  1. Закрепление знаний

  1. Связь между какими физическими величинами устанавливает закон Ома для участка цепи?

  2. Может ли в проводнике протекать ток: а) очень большой силы при малом напряжении; б) малой силы при большом напряжении?

  1. Рефлексия

Бистро ли я усваиваю определения и формулы? Сколько раз я повторяю то, что нужно хорошо знать?

Работа и мощность тока | Физика

Какую работу совершает электрический ток, проходя по тому или иному участку цепи? Чтобы определить это, вспомним, что такое напряжение. Согласно формуле (11.1) U = A/q. Отсюда следует, что

A = qU,     (18.1)

где A — работа тока; q — электрический заряд, прошедший за данное время через рассматриваемый участок цепи. Подставляя в последнее равенство выражение q = It, получаем

A = IUt.     (18.2)

Итак, чтобы найти работу тока на участке цепи, надо напряжение на концах этого участка U умножить на силу тока I и на время t, в течение которого совершалась работа.

Действие тока характеризуют не только работой A, но и мощностью P. Мощность тока показывает, какую работу совершает ток за единицу времени. Если за время t была совершена работа A, то мощность тока P = A/t. Подставляя в это равенство выражение (18.2), получаем

P = IU.      (18.3)

Итак, чтобы найти мощность электрического тока P, надо силу тока I умножить на напряжение U.

В Международной системе единиц (СИ) работу выражают в джоулях (Дж), мощность — в ваттах (Вт), а время — в секундах (с). При этом

1 Вт = 1 Дж/с, 1 Дж = 1 Вт · с.

Мощности некоторых электроустройств, выраженные в киловаттах (1 кВт = 1000 Вт), приведены в таблице 5.

Рассчитаем наибольшую допустимую мощность потребителей электроэнергии, которые могут одновременно работать в квартире. Так как в жилых зданиях сила тока в проводке не должна превышать I = 10 А, то при напряжении U = 220 В соответствующая электрическая мощность оказывается равной:

P = 10 A · 220 В = 2200 Вт = 2,2 кВт.

Одновременное включение в сеть приборов с большей суммарной мощностью приведет к увеличению силы тока и потому недопустимо.

В быту работу тока (или израсходованную на совершение этой работы электроэнергию) измеряют с помощью специального прибора, называемого электрическим счетчиком (счетчиком электроэнергии). При прохождении тока через этот счетчик внутри его начинает вращаться легкий алюминиевый диск. Скорость его вращения оказывается пропорциональной силе тока и напряжению. Поэтому по числу оборотов, сделанных им за данное время, можно судить о работе, совершенной током за это время. Работа тока при этом выражается обычно в киловатт-часах (кВт·ч).

1 кВт·ч — это работа, совершаемая электрическим током мощностью 1 кВт в течение 1 ч. Так как 1 кВт = 1000 Вт, а 1 ч = 3600 с, то

1 кВт·ч = 1000 Вт · 3600 с = 3 600 000 Дж.

??? 1. Как находится работа электрического тока? 2. По какой формуле находится мощность тока? 3. С помощью какого прибора измеряют работу тока? Какая единица работы при этом используется? 4. Сложите мощности всех имеющихся у вас дома электрических устройств. Допустимо ли их одновременное включение в сеть? Почему?

Экспериментальное задание. Рассмотрите у себя дома счетчик электроэнергии. Выясните, как снимаются с него показания. Измерьте с его помощью электроэнергию, израсходованную задень. В течение следующего дня старайтесь экономить энергию — не оставляйте включенным свет, если это не нужно; выключайте электроприборы, которыми в данный момент не пользуетесь; не смотрите все подряд по телевизору. После этого определите с помощью счетчика, сколько электроэнергии вам удалось сэкономить. Вычислите стоимость этой энергии. Сколько денег вам удастся сберечь при подобной экономии энергии за месяц?

Мощность электрического тока — Основы электроники

Обычно электрический ток сравнивают с течением жид­кости по трубке, а напряжение или разность потенциалов — с разностью уровней жидкости.

В этом случае поток воды, падающий сверху вниз, несет с собой определенное количество энергии. В усло­виях свободного падения эта энергия растрачивается беспо­лезно для человека. Если же направить падающий поток во­ды на лопасти турбины, то последняя начнет вращаться и сможет производить полезную работу.

Работа, производимая потоком воды в течение определен­ного промежутка времени, например, в течение одной секун­ды, будет тем больше, чем с большей высоты падает поток и чем больше масса падающей воды.

Точно так же и электрический ток, протекая по цепи от высшего потенциала к низшему, совершает работу. В каждую данную секунду времени будет совершаться тем больше рабо­ты, чем больше разность потенциалов и чем большее количе­ство электричества ежесекундно проходит через поперечное сечение цепи.

Мощность электрического тока это количество работы, совершаемой за одну секунду времени, или скорость совершения работы.

Количество электричества, проходящего через поперечное сечение цепи в течение одной секунды, есть не что иное, как сила тока в цепи. Следовательно, мощность электрического тока будет прямо пропорциональна разности потенциалов (на­пряжению) и силе тока в цепи.

Для измерения мощности электрического тока принята еди­ница, называемая ватт (Вт).

Мощностью в 1 Вт обладает ток силой в 1 А при разности потенциалов, равной 1 В.

Для вычисления мощности постоянного тока в ваттах нуж­но силу тока в амперах умножить на напряжение в вольтах.

Если обозначить мощность электрического тока буквой P, то приведенное выше правило можно записать в виде формулы

P = I*U. (1)

Воспользуемся этой формулой для решения числового при­мера. Требуется определить, какая мощность электрического тока необходима для накала нити радиолампы, если напряжение накала равно 4 в, а ток накала 75 мА

Определим мощность электрического тока, поглощаемую нитью лампы:

Р= 0,075 А*4 В = 0,3 Вт.

Мощность электрического тока можно вычислить и другим путем. Предположим, что нам известны сила тока в цепи и сопротивление цепи, а напряжение неизвестно.

В этом случае мы воспользуемся знакомым нам соотноше­нием из закона Ома:

U=IR

и подставим правую часть этого равенства (IR) в формулу (1) вместо напряжения U.

Тогда формула (1) примет вид:

P = I*U =I*IR

или

Р = I2*R. (2)

Например, требуется узнать, какая мощность теряется в реостате сопротивлением в 5 Ом, если через него проходит ток, силой 0,5 А. Пользуясь формулой (2), найдем:

P= I2*R = (0,5)2*5 =0,25*5 = 1,25 Вт.

Наконец, мощность электрического тока может быть вычислена и в том слу­чае, когда известны напряжение и сопротивление, а сила тока неизвестна. Для этого вместо силы тока I в формулу (1) подставляется известное из закона Ома отношение U/R и тогда формула (1) приобретает следующий вид:

Р = I*U=U2/R (3)

Например, при 2,5 В падения напряжения на реостате сопро­тивлением в 5 Ом поглощаемая реостатом мощность будет равна:

Р = U2/R=(2,5)2/5=1,25 Вт

Таким образом, для вычисления мощности требуется знать любые две из величин, входящих в формулу закона Ома.

Мощность электрического тока равна работе электрического тока, производимой в течение одной секунды.

P = A/t

ПОНРАВИЛАСЬ СТАТЬЯ? ПОДЕЛИСЬ С ДРУЗЬЯМИ В СОЦИАЛЬНЫХ СЕТЯХ!

Похожие материалы:

Добавить комментарий

Как повысить силу тока, не изменяя напряжения

Из статьи вы узнаете как повысить силу тока в цепи зарядного устройства, в блоке питания, трансформатора, в генераторе, в USB портах компьютера не изменяя напряжения.

Что такое сила тока?

Электрический ток представляет собой упорядоченное перемещение заряженных частиц внутри проводника при обязательном наличии замкнутого контура.

Появление тока обусловлено движением электронов и свободных ионов, имеющих положительный заряд.

В процессе перемещения заряженные частицы могут нагревать проводник и оказывать химическое действие на его состав. Кроме того, ток может оказывать влияние на соседние токи и намагниченные тела.

Сила тока — электрический параметр, представляющий собой скалярную величину. Формула:

I=q/t, где I — сила тока, t — время, а q — заряд.

Стоит знать и закон Ома, по которому ток прямо пропорционален U (напряжению) и обратно пропорционален R (сопротивлению).

I=U/R.

Сила тока бывает двух видов — положительной и отрицательной.

Ниже рассмотрим, от чего зависит этот параметр, как повысить силу тока в цепи, в генераторе, в блоке питания и в трансформаторе.

Приведем проверенные рекомендации, которые позволят решить поставленные задачи.

От чего зависит сила тока?

Чтобы повысить I в цепи, важно понимать, какие факторы могут влиять на этот параметр. Здесь можно выделить зависимость от:

  • Сопротивления. Чем меньше параметр R (Ом), тем выше сила тока в цепи.
  • Напряжения. По тому же закону Ома можно сделать вывод, что при росте U сила тока также растет.
  • Напряженности магнитного поля. Чем она больше, тем выше напряжение.
  • Числа витков катушки. Чем больше этот показатель, тем больше U и, соответственно, выше I.
  • Мощности усилия, которое передается на ротор.
  • Диаметра проводников. Чем он меньше, тем выше риск нагрева и перегорания питающего провода.
  • Конструкции источника питания.
  • Диаметра проводов статора и якоря, числа ампер-витков.
  • Параметров генератора — рабочего тока, напряжения, частоты и скорости.

Как повысить силу тока в цепи?

Бывают ситуации, когда требуется повысить I, который протекает в цепи, но при этом важно понимать, что нужно принять меры по защите электроприборов, сделать это можно с помощью специальных устройств.

Рассмотрим, как повысить силу тока с помощью простых приборов.

Для выполнения работы потребуется амперметр.

Вариант 1.

По закону Ома ток равен напряжению (U), деленному на сопротивление (R). Простейший путь повышения силы I, который напрашивается сам собой — увеличение напряжения, которое подается на вход цепи, или же снижение сопротивления. При этом I будет увеличиваться прямо пропорционально U.

К примеру, при подключении цепи в 20 Ом к источнику питания c U = 3 Вольта, величина тока будет равна 0,15 А.

Если добавить к цепи еще один источник питания на 3В, общую величину U удается повысить до 6 Вольт. Соответственно, ток также вырастет в два раза и достигнет предела в 0,3 Ампера.

Подключение источников питания должно осуществляться последовательно, то есть плюс одного элемента подключается к минусу первого.

Для получения требуемого напряжения достаточно соединить в одну группу несколько источников питания.

В быту источники постоянного U, объединенные в одну группу, называются батарейками.

Несмотря на очевидность формулы, практические результаты могут отличаться от теоретических расчетов, что связано с дополнительными факторами — нагревом проводника, его сечением, применяемым материалом и так далее.

В итоге R меняется в сторону увеличения, что приводит и к снижению силы I.

Повышение нагрузки в электрической цепи может стать причиной перегрева проводников, перегорания или даже пожара.

Вот почему важно быть внимательным при эксплуатации приборов и учитывать их мощность при выборе сечения.

Величину I можно повысить и другим путем, уменьшив сопротивление. К примеру, если напряжение на входе равно 3 Вольта, а R 30 Ом, то по цепи проходит ток, равный 0,1 Ампер.

Если уменьшить сопротивление до 15 Ом, сила тока, наоборот, возрастет в два раза и достигнет 0,2 Ампер. Нагрузка снижается почти к нулю при КЗ возле источника питания, в этом случае I возрастают до максимально возможной величины (с учетом мощности изделия).

Дополнительное снизить сопротивление можно путем охлаждения провода. Такой эффект сверхпроводимости давно известен и активно применяется на практике.

Чтобы повысить силу тока в цепи часто применяются электронные приборы, например, трансформаторы тока (как в сварочниках). Сила переменного I в этом случае возрастает при снижении частоты.

Если в цепи переменного тока имеется активное сопротивление, I увеличивается при росте емкости конденсатора и снижении индуктивности катушки.

В ситуации, когда нагрузка имеет чисто емкостной характер, сила тока возрастает при повышении частоты. Если же в цепь входят катушки индуктивности, сила I будет увеличиваться одновременно со снижением частоты.

Также читают — как действует электрический ток на организм человека.

Вариант 2.

Чтобы повысить силу тока, можно ориентироваться на еще одну формулу, которая выглядит следующим образом:

I = U*S/(ρ*l). Здесь нам неизвестно только три параметра:

  • S — сечение провода;
  • l — его длина;
  • ρ — удельное электрическое сопротивление проводника.

Чтобы повысить ток, соберите цепочку, в которой будет источник тока, потребитель и провода.

Роль источника тока будет выполнять выпрямитель, позволяющий регулировать ЭДС.

Подключайте цепочку к источнику, а тестер к потребителю (предварительно настройте прибор на измерение силы тока). Повышайте ЭДС и контролируйте показатели на приборе.

Как отмечалось выше, при росте U удается повысить и ток. Аналогичный эксперимент можно сделать и для сопротивления.

Для этого выясните, из какого материала сделаны провода и установите изделия, имеющие меньшее удельное сопротивление. Если найти другие проводники не удается, укоротите те, что уже установлены.

Еще один путь — увеличение поперечного сечения, для чего параллельно установленным проводам стоит смонтировать аналогичные проводники. В этом случае возрастает площадь сечения провода и увеличивается ток.

Если же укоротить проводники, интересующий нас параметр (I) возрастет. При желании варианты увеличения силы тока разрешается комбинировать. Например, если на 50% укоротить проводники в цепи, а U поднять на 300%, то сила I возрастет в 9 раз.

Как повысить силу тока в блоке питания?

В интернете часто можно встретить вопрос, как повысить I в блоке питания, не изменяя напряжение. Рассмотрим основные варианты.

Ситуация №1.

Блок питания на 12 Вольт работает с током 0,5 Ампер. Как поднять I до предельной величины? Для этого параллельно БП ставится транзистор. Кроме того, на входе устанавливается резистор и стабилизатор.

Узнайте больше — как проверить транзистор мультиметром на исправность.

При падении напряжения на сопротивлении до нужной величины открывается транзистор, и остальной ток протекает не через стабилизатор, а через транзистор.

Последний, к слову, необходимо выбирать по номинальному току и ставить радиатор.

Кроме того, возможны следующие варианты:

  • Увеличить мощность всех элементов устройства. Поставить стабилизатор, диодный мост и трансформатор большей мощности.
  • При наличии защиты по току снизить номинал резистора в цепочке управления.

Ситуация №2.

Имеется блок питания на U = 220-240 Вольт (на входе), а на выходе постоянное U = 12 Вольт и I = 5 Ампер. Задача — увеличить ток до 10 Ампер. При этом БП должен остаться приблизительно в тех же габаритах и не перегреваться.

Здесь для повышения мощности на выходе необходимо задействовать другой трансформатор, который пересчитан под 12 Вольт и 10 Ампер. В противном случае изделие придется перематывать самостоятельно.

При отсутствии необходимого опыта на риск лучше не идти, ведь высока вероятность короткого замыкания или перегорания дорогостоящих элементов цепи.

Трансформатор придется поменять на изделие большего размера, а также пересчитывать цепочку демпфера, находящегося на СТОКЕ ключа.

Следующий момент — замена электролитического конденсатора, ведь при выборе емкости нужно ориентироваться на мощность устройства. Так, на 1 Вт мощности приходится 1-2 мкФ.

Также рекомендуется поменять диоды с выпрямителями. Кроме того, может потребоваться установка нового диода выпрямителя на низкой стороне и увеличение емкости конденсаторов.

После такой переделки устройство будет греться сильнее, поэтому без установки вентилятора не обойтись.

Как повысить силу тока в зарядном устройстве?

В процессе пользования зарядными устройствами можно заметить, что ЗУ для планшета, телефона или ноутбука имеют ряд отличий. Кроме того, может различаться и скорость, с которой происходит заряд девайсов.

Здесь многое зависит от того, используется оригинальное или неоригинальное устройство.

Чтобы измерить ток, который поступает к планшету или телефону от зарядного устройства, можно использовать не только амперметр, но и приложение Ampere.

С помощью софта удается выяснить скорость заряда и разрядки АКБ, а также его состояние. Приложением можно пользоваться бесплатно. Единственным недостатком является реклама (в платной версии ее нет).

Главной проблемой зарядки аккумуляторов является небольшой ток ЗУ, из-за чего время набора емкости слишком большое. На практике ток, протекающий в цепи, напрямую зависит от мощности зарядного устройства, а также других параметров — длины кабеля, его толщины и сопротивления.

С помощью приложения Ampere можно увидеть, при какой силе тока производится заряд девайса, а также проверить, может ли изделие заряжаться с большей скоростью.

Для использования возможностей приложения достаточно скачать его, установить и запустить.

После этого телефон, планшет или другое устройство подключается к зарядному устройству. Вот и все — остается обратить внимание на параметры тока и напряжения.

Кроме того, вам будет доступна информация о типе батареи, уровне U, состоянии АКБ, а также температурном режиме. Также можно увидеть максимальные и минимальные I, имеющие место в период цикла.

Если в распоряжении имеется несколько ЗУ, можно запустить программу и пробовать делать зарядку каждым из них. По результатам тестирования проще сделать выбор ЗУ, обеспечивающего максимальный ток. Чем выше будет этот параметр, тем быстрее зарядится девайс.

Измерение силы тока — не единственное, на что способно приложение Ampere. С его помощью можно проверить, сколько потребляется I в режиме ожидания или при включении различных игр (приложений).

Например, после отключения яркости дисплея, деактивации GPS или передачи данных легко заметить снижение нагрузки. На этом фоне проще сделать вывод, какие опции в большей степени разряжают аккумулятор.

Что еще стоит отметить? Все производители рекомендуют заряжать девайсы «родными» ЗУ, выдающими определенный ток.

Но в процессе эксплуатации бывают ситуации, когда приходится заряжать телефон или планшет другими зарядными, имеющими большую мощность. В итоге скорость зарядки может оказаться выше. Но не всегда.

Мало, кто знает, но некоторые производители ограничивают предельный ток, который может принимать АКБ устройства.

Например, устройство Самсунг Гэлекси Альфа поставляется вместе с зарядным на ток 1,35 Ампер.

При подключении 2-амперного ЗУ ничего не меняется — скорость зарядки осталась той же. Это объясняется ограничением, которое установлено производителем. Аналогичный тест был произведен и с рядом других телефонов, что только подтвердило догадку.

С учетом сказанного выше можно сделать вывод, что «неродные» ЗУ вряд ли причинят вред аккумулятору, но иногда могут помочь в более быстрой зарядке.

Рассмотрим еще одну ситуацию. При зарядке девайса через USB-разъем АКБ набирает емкость медленнее, чем если заряжать устройство от обычного ЗУ.

Это объясняется ограничением силы тока, которую способен отдавать USB порт (не больше 0,5 Ампер для USB 2.0). В случае применения USB3.0 сила тока возрастает до уровня 0,9 Ампер.

Кроме того, существует специальная утилита, позволяющая «тройке» пропускать через себя больший I.

Для устройств типа Apple программа называется ASUS Ai Charger, а для других устройств — ASUS USB Charger Plus.

Как повысить силу тока в трансформаторе?

Еще один вопрос, который тревожит любителей электроники — как повысить силу тока применительно к трансформатору.

Здесь можно выделить следующие варианты:

  • Установить второй трансформатор;
  • Увеличить диаметр проводника. Главное, чтобы позволило сечение «железа».
  • Поднять U;
  • Увеличить сечение сердечника;
  • Если трансформатор работает через выпрямительное устройство, стоит применить изделие с умножителем напряжения. В этом случае U увеличивается, а вместе с ним растет и ток нагрузки;
  • Купить новый трансформатор с подходящим током;
  • Заменить сердечник ферромагнитным вариантом изделия (если это возможно).

В трансформаторе работает пара обмоток (первичная и вторичная). Многие параметры на выходе зависят от сечения проволоки и числа витков. Например, на высокой стороне X витков, а на другой — 2X.

Это значит, что напряжение на вторичной обмотке будет ниже, как и мощность. Параметр на выходе зависит и от КПД трансформатора. Если он меньше 100%, снижается U и ток во вторичной цепи.

С учетом сказанного выше можно сделать следующие выводы:

  • Мощность трансформатора зависит от ширины постоянного магнита.
  • Для увеличения тока в трансформаторе требуется снижение R нагрузки.
  • Ток (А) зависит от диаметра обмотки и мощности устройства.
  • В случае перемотки рекомендуется использовать провод большей толщины. При этом отношение провода по массе на первичной и вторичной обмотке приблизительно идентично. Если на первичную обмотку намотать 0,2 кг железа, а на вторичную — 0,5 кг, первичка сгорит.

Как повысить силу тока в генераторе?

Ток в генераторе напрямую зависит от параметра сопротивления нагрузки. Чем ниже этот параметр, тем выше ток.

Если I выше номинального параметра, это свидетельствует о наличии аварийного режима — уменьшения частоты, перегрева генератора и прочих проблем.

Для таких случаев должна быть предусмотрена защита или отключение устройства (части нагрузки).

Кроме того, при повышенном сопротивлении напряжение снижается, происходит подсадка U на выходе генератора.

Чтобы поддерживать параметр на оптимальном уровне, обеспечивается регулирование тока возбуждения. При этом повышение тока возбуждения ведет к росту напряжения генератора.

Частота сети должна находиться на одном уровне (быть постоянной величиной).

Рассмотрим пример. В автомобильном генераторе необходимо повысить ток с 80 до 90 Ампер.

Для решения этой задачи требуется разобрать генератор, отделить обмотку и припаять к ней вывод с последующим подключением диодного моста.

Кроме того, сам диодный мост меняется на деталь большей производительности.

После этого требуется снять обмотку и кусок изоляции в месте, где должен припаиваться провод.

При наличии неисправного генератора с него откусывается вывод, после чего с помощью медной проволоки наращиваются ножки такой же толщины.

После припаивания место стыка изолируется термоусадкой.

Следующим этапом требуется купить 8-диодный мост. Найти его — весьма сложная задача, но нужно постараться.

Перед установкой желательно проверить изделие на исправность (если деталь б/у, возможен пробой одного или нескольких диодов).

После установки моста крепите конденсатор, а далее — регулятор напряжения на 14,5 Вольт.

Можно приобрести пару регуляторов — на 14,5 (немецкий) и на 14 Вольт (отечественный).

Теперь высверливаются клепки, отпаиваются ножки и разделяются таблетки. Далее таблетка подпаивается к отечественному регулятору, который фиксируется с помощью винтов.

Остается припаять отечественную «таблетку» к иностранному регулятору и собирать генератор.

Итоги

Как видно из статьи, повысить силу тока, не изменяя напряжение в сети, реально.

Главное — разобраться с особенностями конструкции устройства, которое подлежит корректировке, и иметь практические навыки работы с измерительными приборами и паяльником. Кроме того, важно осознавать потенциальные риски от внесения корректировок.

опаснее для человека Вольты или Амперы

Всем известно, что электричество опасно для здоровья и жизни людей. Об этом рассказывают в школе, на это указывают предупреждающие надписи та высоковольтных трансформаторах «Опасно для жизни, высокое напряжение!» и на розетках «220В».

Однако в ПТБЭЭП и других нормативных документах кроме напряжения указывается опасный ток. Даже УЗО и дифференциальные автоматы защищают не от попадания человека под напряжение, а от протекания через него тока, превышающего ток уставки. Так что же представляет бОльшую опасность и что убивает ток или напряжение?

Как возникает ток и напряжение

Для ответа на вопрос, что убивает ток или напряжение, необходимо разобраться, к каким физическим явлениям относятся эти термины. Несмотря на то, что они связаны между собой, это два разных понятия.

Что такое электрический ток

Согласно школьному курсу физики и Теоретическим Основам Электротехники (ТОЭ) электрическим током называется направленное движение электрических частиц. В металлах это электроны, а в жидкостях, в том числе организме человека, ионы солей, кислот и щелочей. Именно поэтому дистиллированная вода является изолятором.

Единицей измерения является 1 Ампер. Это около 6,24 × 1018 электронов, протекающих через проводник за 1 секунду.

Интересно! Воздействие токов небольшой величины применяются в медицине в установках УВЧ и для лечения некоторых заболеваний.

Что такое напряжение

Электрическое напряжение — это разность потенциалов между двумя точками или проводами. Этот потенциал приводит в движение заряженные частицы и вызывает появление электрического тока в проводнике. Говоря об опасном токе и напряжение для человека чаще всего подразумевается один из проводов и заземление.

При наличии только одного контакта разность потенциалов и напряжение отсутствует. Именно поэтому птицы могут сидеть на высоковольтных проводах, а сама линия электропередач монтируется так, чтобы исключить одновременное прикосновение пернатых к двум проводам или к проводу и опоре.

Отличие между током и напряжением

Различие между током и напряжением проще всего показать на примере водопровода и водонапорной башни. В данной системе аналогом напряжения является высота башни и давление в системе, а ток — это поток воды в трубах.

Чем выше башня и давление (напряжение) и больше сечение (меньше электрическое сопротивление), тем больше поток воды (ток).

Кроме того, напряжение как потенциал может существовать неопределённо долго, а ток протекает только при замкнутой цепи между точками с различным потенциалом.

Справка! Мощность электроприбора рассчитывается произведением тока и напряжения.

Воздействие тока и напряжения на организм

Для появления тока к проводнику необходимо подать напряжение и ток тем больше, чем оно выше. С точки зрения электротехники тело человека является раствором солей и других химических веществ в воде и ток, протекающий через него, так же подчиняется этому правилу, определяющему, что убивает человека сила тока или напряжение.

Протекание через организм человека электрического тока оказывает различные виды негативных воздействий:

  • термическое — нагрев организма по пути протекания, а при большой величине тока ожоги;
  • электролитическое — различные химические реакции в крови и биологических жидкостях;
  • биологическое — раздражение нервных окончаний в коже и других органах;
  • механическое — разрывы, вывихи и расслоения тканей из-за электродинамического эффекта.

Сами электротравмы делятся на общие, при которых поражается весь организм, и местные, при которых негативному воздействию подвергаются только отдельные участки кожи и ожоги глаз ультрафиолетовым излучением электрической дуги.

От чего зависит степень поражения

То, какое напряжение и ток опасны для жизни, зависит от различных факторов, главный из которых электрическое сопротивление кожи. Если её поверхность сухая и чистая, то сопротивление при напряжении 5-10В составляет около 100кОм, а при намокании оно падает до 1кОм. Его так же уменьшают порезы и царапины. Сопротивление внутренних органов 0,5-1кОм.

Сопротивление тела падает, а протекающий через организм ток растёт при увеличении напряжения, продолжительности воздействия, плохом состоянии здоровья и других факторах. При совпадении всех негативных факторов оно может понизиться до 0,8кОм.

Кроме напряжения степень поражения зависит так же от длительности и пути прохождения тока через организм. Самым опасным является путь прохождения тока рука-рука и рука-ноги, при которых ток проходит через область груди.

Чем выше напряжение и ток, тем меньше относительно безопасное время его протекания:

  • 65В — 1с;
  • 220В — 0,1с.

При более продолжительном нахождении человека под напряжением возрастает вероятность фибрилляции желудочков сердца с его последующей остановкой. В этом случае спасти жизнь пострадавшему могут только искусственное дыхание и непрямой массаж сердца.

Важно! Реанимационные действия производятся только после освобождения человека от воздействия электричества.

Опасный ток и напряжение для человека

Величина опасного для здоровья и жизни тока зависит, прежде всего, от рода тока — постоянный или переменный:

  • Постоянный ток менее опасен, ощущается при 12мА. Взявшись рукой за провод, находящийся под напряжением, его можно самостоятельно отпустить при токе до 25мА, остановка дыхания наступает при 110мА.
  • Переменный ток промышленной частоты более опасен. Ощущается при 0,6мА, причиняет боль при 15мА, при 50мА останавливается дыхание, смертельным является ток 90мА.
  • Переменный ток высокой частоты. Распространяется по поверхности тела, вызывает ожоги кожи, но не повреждает внутренние органы.

Самым высоким сопротивлением обладает верхний ороговевший слой сухой кожи. При низких напряжениях он составляет 40-100кОм, но при повышении происходит электрический пробой изоляции и сопротивление тела падает до 1кОм.

Оно так же понижается во влажных помещениях, поэтому максимально-допустимое напряжение в парных и саунах составляет 12В.

Понизить сопротивление поверхности тела может так же находящиеся на ней пот, загрязнения и другие факторы, в результате опасным может быть напряжение 50В. Поэтому питание переносных светильников ограничено величиной 36В.

При рассмотрении вопроса, что убивает сила тока или напряжение, необходимо учесть, что статическое электричество, за исключением специальных установок типа «лейденской банки» совершенно безопасно.

В бытовых условиях человек с ним сталкивается при ношении шерстяного свитера или поглаживании кошки. Его величина может достигать 35кВ, но из-за малой величины заряда ощущается как кратковременный укол. Это относится так же к пьезоподжигу в карманных зажигалках.

Вывод

Как видно из статьи, ответ на вопрос, что убивает ток или напряжение, не является однозначным. С одной стороны, без напряжения электрический ток отсутствует, а с другой стороны, само по себе высокое напряжение не опасно и при разомкнутой цепи, в том числе через тело человека, ток отсутствует.

Поэтому, несмотря на то, что убивает именно ток, опасным является высокое напряжение.

Похожие материалы на сайте:

Понравилась статья — поделись с друзьями!

 

Напряжение и ток | Основные понятия электричества

Как упоминалось ранее, нам нужно нечто большее, чем просто непрерывный путь (т. Так же, как мрамор в трубе или вода в трубе, для инициирования потока требуется некоторая сила воздействия. В случае электронов эта сила — это та же сила, которая действует в статическом электричестве: сила, создаваемая дисбалансом электрического заряда.Если мы возьмем примеры воска и шерсти, которые были натерты друг с другом, мы обнаружим, что избыток электронов в воске (отрицательный заряд) и дефицит электронов в шерсти (положительный заряд) создают дисбаланс заряда между ними. Этот дисбаланс проявляется как сила притяжения между двумя объектами:

Если между заряженным воском и шерстью поместить токопроводящую проволоку, электроны будут проходить через нее, так как некоторые из избыточных электронов в воске устремляются через провод, чтобы вернуться к шерсти, восполняя там недостаток электронов:

Дисбаланс электронов между атомами воска и атомами шерсти создает силу между двумя материалами.Поскольку электроны не могут перетекать от воска к шерсти, все, что может сделать эта сила, — это притягивать два объекта вместе. Однако теперь, когда проводник перекрывает изолирующий зазор, сила заставит электроны течь в однородном направлении через провод, хотя бы на мгновение, пока заряд в этой области не нейтрализуется и сила между воском и шерстью не уменьшится. Электрический заряд, образованный между этими двумя материалами при трении их друг о друга, служит для хранения определенного количества энергии. Эта энергия мало чем отличается от энергии, накопленной в высоком резервуаре с водой, который выкачивается из пруда нижнего уровня:

Влияние силы тяжести на воду в резервуаре создает силу, которая пытается снова опустить воду на более низкий уровень.Если подходящая труба будет проложена от резервуара обратно к пруду, вода под действием силы тяжести потечет вниз из резервуара по трубе:

Для перекачки этой воды из пруда с низким уровнем в резервуар с высоким уровнем требуется энергия, и движение воды по трубопроводу обратно к исходному уровню представляет собой высвобождение энергии, накопленной от предыдущей откачки. Если вода перекачивается на еще более высокий уровень, для этого потребуется еще больше энергии, таким образом, будет сохранено больше энергии, и больше энергии будет высвобождено, если воде позволить снова течь по трубе обратно вниз:

Электроны мало чем отличаются.Если мы протираем воск и шерсть вместе, мы «выкачиваем» электроны с их нормальных «уровней», создавая условия, при которых существует сила между парафином и шерстью, поскольку электроны стремятся восстановить свои прежние положения (и балансировать внутри своего тела). соответствующие атомы). Сила, притягивающая электроны обратно в их исходное положение вокруг положительных ядер их атомов, аналогична силе гравитации, действующей на воду в резервуаре, пытаясь вернуть ее к прежнему уровню. Подобно тому, как перекачка воды на более высокий уровень приводит к накоплению энергии, «перекачка» электронов для создания дисбаланса электрического заряда приводит к накоплению определенного количества энергии в этом дисбалансе.И точно так же, как предоставление возможности воде стекать обратно с высоты резервуара приводит к высвобождению этой накопленной энергии, предоставление возможности электронам течь обратно к их первоначальным «уровням» приводит к высвобождению накопленной энергии. Когда носители заряда находятся в этом статическом состоянии (точно так же, как вода, неподвижная, высоко в резервуаре), энергия, хранящаяся там, называется потенциальной энергией , потому что у нее есть возможность (потенциал) высвобождения, которая не была полностью реализована. еще.

Понимание концепции напряжения

Когда носители заряда находятся в этом статическом состоянии (точно так же, как вода, неподвижная, высоко в резервуаре), энергия, хранящаяся там, называется потенциальной энергией, потому что у нее есть возможность (потенциал) высвобождения, которая еще не полностью реализована. . Когда вы терзаете обувь с резиновой подошвой о тканевый ковер в сухой день, вы создаете дисбаланс электрического заряда между вами и ковром. При царапании ногами накапливается энергия в виде дисбаланса зарядов, вытесняемых из их первоначальных мест.Этот заряд (статическое электричество) является стационарным, и вы вообще не заметите, что энергия накапливается. Однако, как только вы положите руку на металлическую дверную ручку (с большой подвижностью электронов для нейтрализации вашего электрического заряда), эта накопленная энергия будет высвобождена в виде внезапного потока заряда через вашу руку, и вы будете воспринимать ее как поражение электрическим током! Эта потенциальная энергия, хранящаяся в виде дисбаланса электрического заряда и способная провоцировать протекание носителей заряда через проводник, может быть выражена термином, называемым напряжением, которое технически является мерой потенциальной энергии на единицу заряда или чего-то, что физик мог бы называют удельной потенциальной энергией.

Определение напряжения

Определяемое в контексте статического электричества, напряжение — это мера работы, необходимой для перемещения единичного заряда из одного места в другое, против силы, которая пытается сохранить баланс электрических зарядов. В контексте источников электроэнергии напряжение — это количество доступной потенциальной энергии (работа, которую необходимо выполнить) на единицу заряда для перемещения зарядов через проводник, поскольку напряжение — это выражение потенциальной энергии, представляющее возможность или потенциал для выделения энергии когда заряд перемещается с одного «уровня» на другой, он всегда находится между двумя точками.Рассмотрим аналогию с водохранилищем:

.

Из-за разницы в высоте падения существует вероятность того, что гораздо больше энергии будет выпущено из резервуара через трубопровод в точку 2, чем в точку 1. Принцип интуитивно понятен при падении камня: что приводит к более сильный удар, камень упал с высоты одного фута или тот же камень упал с высоты одной мили? Очевидно, что падение с большей высоты приводит к высвобождению большей энергии (более сильному удару).Мы не можем оценить количество запасенной энергии в водном резервуаре, просто измерив объем воды, точно так же, как мы можем предсказать серьезность удара падающей породы, просто зная вес породы: в обоих случаях мы также должны учитывать, как далекие эти массы упадут со своей начальной высоты. Количество энергии, высвобождаемой при падении массы, зависит от расстояния между его начальной и конечной точками. Точно так же потенциальная энергия, доступная для перемещения носителей заряда из одной точки в другую, зависит от этих двух точек.Следовательно, напряжение всегда выражается как величина между двумя точками . Достаточно интересно, что аналогия с массой, потенциально «падающей» с одной высоты на другую, является настолько удачной моделью, что напряжение между двумя точками иногда называют падением напряжения .

Генерирующее напряжение

Напряжение можно генерировать другими способами, кроме трения материалов определенных типов друг о друга. Химические реакции, лучистая энергия и влияние магнетизма на проводники — вот несколько способов создания напряжения.Соответствующими примерами этих трех источников напряжения являются батареи, солнечные элементы и генераторы (например, «генератор переменного тока» под капотом вашего автомобиля). На данный момент мы не будем вдаваться в подробности того, как работает каждый из этих источников напряжения — более важно то, что мы понимаем, как источники напряжения могут применяться для создания потока заряда в электрической цепи. Давайте возьмем символ химической батареи и шаг за шагом построим схему:

Как работают источники напряжения?

Любой источник напряжения, включая аккумуляторные батареи, имеет две точки электрического контакта.В этом случае у нас есть точка 1 и точка 2 на приведенной выше диаграмме. Горизонтальные линии разной длины указывают на то, что это батарея, и дополнительно указывают направление, в котором напряжение этой батареи будет пытаться протолкнуть носители заряда по цепи. Тот факт, что горизонтальные линии в символе батареи кажутся разделенными (и, таким образом, не могут служить в качестве пути для потока заряда), не вызывает беспокойства: в реальной жизни эти горизонтальные линии представляют собой металлические пластины, погруженные в жидкий или полутвердый материал. который не только проводит заряды, но и генерирует напряжение, чтобы подтолкнуть их, взаимодействуя с пластинами.Обратите внимание на маленькие значки «+» и «-» непосредственно слева от символа батареи. Отрицательный (-) конец батареи всегда является концом с самым коротким тире, а положительный (+) конец батареи всегда является концом с самым длинным тире. Положительный конец батареи — это конец, который пытается вытолкнуть из нее носители заряда (помните, что по традиции мы думаем, что носители заряда заряжены положительно, хотя электроны заряжены отрицательно). Точно так же отрицательный конец — это конец, который пытается привлечь носители заряда.Когда «+» и «-» концы батареи ни к чему не подключены, между этими двумя точками будет напряжение, но не будет потока заряда через батарею, потому что нет непрерывного пути, по которому могут перемещаться носители заряда.

Тот же принцип справедлив и для аналогии с резервуаром для воды и насосом: без возвратной трубы обратно в пруд накопленная энергия в резервуаре не может быть выпущена в виде потока воды. Когда резервуар полностью заполнен, поток не может возникнуть, независимо от того, какое давление может создать насос.Должен существовать полный путь (контур), по которому вода течет из пруда в резервуар и обратно в пруд для обеспечения непрерывного потока. Мы можем обеспечить такой путь для батареи, соединив кусок провода от одного конца батареи к другому. Образуя цепь с петлей из проволоки, мы инициируем непрерывный поток заряда по часовой стрелке:

Понимание концепции электрического тока

Пока батарея продолжает вырабатывать напряжение и непрерывность электрического пути не нарушена, носители заряда будут продолжать течь в цепи.Следуя метафоре воды, движущейся по трубе, этот непрерывный, равномерный поток заряда через цепь называется током . Пока источник напряжения продолжает «толкать» в одном направлении, носители заряда будут продолжать двигаться в том же направлении в цепи. Этот однонаправленный поток тока называется постоянным током, или постоянным током. Во втором томе этой серии книг исследуются электрические цепи, в которых направление тока переключается взад и вперед: переменного тока, или переменного тока.Но пока мы просто займемся цепями постоянного тока. Поскольку электрический ток состоит из отдельных носителей заряда, протекающих в унисон через проводник, перемещаясь и толкая носители заряда впереди, точно так же, как шарики через трубу или вода через трубу, величина потока в одной цепи будет одинаковой в любой момент. Если бы мы отслеживали поперечное сечение провода в одной цепи, считая протекающие носители заряда, мы бы заметили точно такое же количество в единицу времени, что и в любой другой части цепи, независимо от длины проводника или проводника. диаметр.Если мы нарушим непрерывность цепи в любой точке , электрический ток прекратится во всем контуре, и полное напряжение, создаваемое батареей, будет проявляться через разрыв между концами проводов, которые раньше были соединены:

Что такое полярность падения напряжения?

Обратите внимание на знаки «+» и «-», нарисованные на концах разрыва цепи, и то, как они соответствуют знакам «+» и «-» рядом с выводами аккумулятора. Эти маркеры указывают направление, в котором напряжение пытается протолкнуть ток, это направление потенциала, обычно называемое полярностью , .Помните, что напряжение всегда относительно между двумя точками. По этой причине полярность падения напряжения также является относительной между двумя точками: будет ли точка в цепи помечена знаком «+» или «-», зависит от другой точки, к которой она относится. Взгляните на следующую схему, где каждый угол петли отмечен номером для справки:

При нарушении целостности цепи между точками 2 и 3 полярность падения напряжения между точками 2 и 3 будет «+» для точки 2 и «-» для точки 3.Полярность батареи (1 «+» и 4 «-») пытается протолкнуть ток через петлю по часовой стрелке от 1 до 2, от 3 до 4 и снова обратно к 1. Теперь давайте посмотрим, что произойдет, если мы снова соединим точки 2 и 3 вместе, но сделаем разрыв цепи между точками 3 и 4:

При разрыве между 3 и 4 полярность падения напряжения между этими двумя точками будет «-» для 4 и «+» для 3. Обратите особое внимание на тот факт, что «знак» точки 3 противоположен знаку в Первый пример, где разрыв был между точками 2 и 3 (где точка 3 была помечена «-»).Мы не можем сказать, что точка 3 в этой цепи всегда будет либо «+», либо «-», потому что полярность, как и само напряжение, не зависит от одной точки, а всегда относительна между двумя точками!

ОБЗОР:

  • Носители заряда могут двигаться через проводник с помощью той же силы, которая проявляется в статическом электричестве.
  • Напряжение — это мера удельной потенциальной энергии (потенциальной энергии на единицу заряда) между двумя точками.С точки зрения непрофессионала, это мера «толчка», позволяющая мотивировать обвинение.
  • Напряжение, как выражение потенциальной энергии, всегда относительно между двумя местоположениями или точками. Иногда это называют «падением напряжения».
  • Когда источник напряжения подключен к цепи, напряжение вызывает равномерный поток носителей заряда через эту цепь, называемый током .
  • В одиночной (однопетлевой) схеме величина тока в любой точке такая же, как и величина тока в любой другой точке.
  • Если цепь, содержащая источник напряжения, разорвана, полное напряжение этого источника появится в точках разрыва.
  • +/- ориентация падения напряжения называется полярностью . Это также относительное значение между двумя точками.

СВЯЗАННЫЕ РАБОЧИЕ ЛИСТЫ:

Закон

Ома — Как соотносятся напряжение, ток и сопротивление | Закон Ома

Первая и, возможно, самая важная взаимосвязь между током, напряжением и сопротивлением называется законом Ома, который был открыт Георгом Симоном Омом и опубликован в его статье 1827 года «Гальваническая цепь, исследованная математически».

Напряжение, ток и сопротивление

Электрическая цепь образуется, когда создается проводящий путь, позволяющий электрическому заряду непрерывно перемещаться. Это непрерывное движение электрического заряда через проводники цепи называется током , и его часто называют «потоком», как поток жидкости через полую трубу.

Сила, побуждающая носители заряда «течь» в цепи, называется напряжением . Напряжение — это особая мера потенциальной энергии, которая всегда относительна между двумя точками.

Когда мы говорим об определенном количестве напряжения, присутствующем в цепи, мы имеем в виду измерение того, сколько потенциальной энергии существует для перемещения носителей заряда из одной конкретной точки в этой цепи в другую конкретную точку. Без ссылки на , две конкретные точки , термин «напряжение» не имеет значения.

Ток имеет тенденцию проходить через проводники с некоторой степенью трения или сопротивления движению. Это противодействие движению правильнее называть сопротивлением .Сила тока в цепи зависит от величины напряжения и величины сопротивления в цепи, препятствующей прохождению тока.

Как и напряжение, сопротивление — это величина, относительная между двумя точками. По этой причине величины напряжения и сопротивления часто указываются как «между» или «поперек» двух точек в цепи.

Единицы измерения: вольт, ампер и ом

Чтобы иметь возможность делать осмысленные утверждения об этих величинах в цепях, мы должны уметь описывать их количества так же, как мы могли бы количественно определить массу, температуру, объем, длину или любой другой вид физической величины.Для массы мы можем использовать единицы «килограмм» или «грамм».

Для температуры мы можем использовать градусы Фаренгейта или градусы Цельсия. Вот стандартные единицы измерения электрического тока, напряжения и сопротивления:

«Символ», присвоенный каждой величине, представляет собой стандартную буквенную букву, используемую для представления этой величины в алгебраическом уравнении. Подобные стандартизированные буквы распространены в физических и технических дисциплинах и признаны во всем мире.

«Аббревиатура единицы» для каждой величины представляет собой алфавитный символ, используемый в качестве сокращенного обозначения для конкретной единицы измерения. И да, этот странно выглядящий символ «подкова» — это заглавная греческая буква Ω, просто символ иностранного алфавита (извинения перед читателями-греками).

Каждая единица измерения названа в честь известного экспериментатора в области электричества: amp в честь француза Андре М. Ампера, вольт в честь итальянца Алессандро Вольта и Ом в честь немца Георга Симона Ома.

Математический символ для каждой величины также имеет значение. «R» для сопротивления и «V» для напряжения говорят сами за себя, тогда как «I» для тока кажется немного странным. Считается, что буква «I» должна представлять «интенсивность» (потока заряда), а другой символ напряжения, «E», означает «электродвижущую силу». Судя по исследованиям, которые мне удалось провести, кажется, что есть некоторые споры по поводу значения слова «я».

Символы «E» и «V» по большей части взаимозаменяемы, хотя в некоторых текстах зарезервировано «E» для обозначения напряжения на источнике (таком как батарея или генератор) и «V» для обозначения напряжения на чем-либо еще.

Все эти символы выражаются заглавными буквами, за исключением случаев, когда величина (особенно напряжение или ток) описывается в терминах короткого периода времени (называемого «мгновенным» значением). Например, напряжение батареи, которое стабильно в течение длительного периода времени, будет обозначаться заглавной буквой «E», в то время как пик напряжения при ударе молнии в тот самый момент, когда он попадает в линию электропередачи, скорее всего, будет обозначается строчной буквой «е» (или строчной буквой «v»), чтобы обозначить это значение как имеющееся в один момент времени.

То же самое соглашение о нижнем регистре справедливо и для тока, строчная буква «i» представляет ток в некоторый момент времени. Однако большинство измерений постоянного тока (DC), которые стабильны во времени, будут обозначены заглавными буквами.

Кулон и электрический заряд

Одна из основополагающих единиц электрического измерения, которую часто преподают в начале курсов электроники, но нечасто используют впоследствии, — это единица кулонов , которая представляет собой меру электрического заряда, пропорционального количеству электронов в несбалансированном состоянии.Один кулон заряда равен 6 250 000 000 000 000 000 электронов.

Символом количества электрического заряда является заглавная буква «Q», а единица измерения кулонов обозначается заглавной буквой «C». Бывает так, что единица измерения тока, ампер, равна 1 кулону заряда, проходящему через заданную точку в цепи за 1 секунду. В этих терминах ток — это скорость движения электрического заряда по проводнику.

Как указывалось ранее, напряжение является мерой потенциальной энергии на единицу заряда , доступной для стимулирования протекания тока из одной точки в другую.Прежде чем мы сможем точно определить, что такое «вольт», мы должны понять, как измерить эту величину, которую мы называем «потенциальной энергией». Общая метрическая единица для энергии любого вида — джоуль, , что равняется количеству работы, совершаемой силой в 1 ньютон при движении на 1 метр (в том же направлении).

В британских подразделениях это чуть меньше 3/4 фунта силы, приложенной на расстоянии 1 фута. Проще говоря, требуется около 1 джоуля энергии, чтобы поднять гирю весом 3/4 фунта на 1 фут от земли или перетащить что-то на расстояние 1 фут, используя параллельную тяговую силу 3/4 фунта.В этих научных терминах 1 вольт равен 1 джоулю электрической потенциальной энергии на (деленный на) 1 кулон заряда. Таким образом, 9-вольтовая батарея выделяет 9 джоулей энергии на каждый кулон заряда, проходящего через цепь.

Эти единицы и символы электрических величин станут очень важны, когда мы начнем исследовать отношения между ними в цепях.

Уравнение закона Ома

Принципиальное открытие

Ома заключалось в том, что величина электрического тока, протекающего через металлический проводник в цепи, прямо пропорциональна напряжению, приложенному к нему при любой заданной температуре.Ом выразил свое открытие в виде простого уравнения, описывающего взаимосвязь напряжения, тока и сопротивления:

В этом алгебраическом выражении напряжение (E) равно току (I), умноженному на сопротивление (R). Используя методы алгебры, мы можем преобразовать это уравнение в два варианта, решая для I и R соответственно:

Анализ простых схем с помощью закона Ома

Давайте посмотрим, как эти уравнения могут работать, чтобы помочь нам анализировать простые схемы:

В приведенной выше схеме есть только один источник напряжения (батарея слева) и только один источник сопротивления току (лампа справа).Это позволяет очень легко применять закон Ома. Если мы знаем значения любых двух из трех величин (напряжения, тока и сопротивления) в этой цепи, мы можем использовать закон Ома для определения третьей.

В этом первом примере мы рассчитаем величину тока (I) в цепи, учитывая значения напряжения (E) и сопротивления (R):

Какой ток (I) в этой цепи?

В этом втором примере мы рассчитаем величину сопротивления (R) в цепи, учитывая значения напряжения (E) и тока (I):

Какое сопротивление (R) дает лампа?

В последнем примере мы рассчитаем величину напряжения, подаваемого батареей, с учетом значений тока (I) и сопротивления (R):

Какое напряжение обеспечивает аккумулятор?

Техника треугольника закона Ома

Закон Ома — очень простой и полезный инструмент для анализа электрических цепей.Он так часто используется при изучении электричества и электроники, что серьезный студент должен запомнить его. Для тех, кто еще не знаком с алгеброй, есть уловка, позволяющая вспомнить, как решить для любого количества, учитывая два других.

Сначала расположите буквы E, I и R в виде треугольника следующим образом:

Если вы знаете E и I и хотите определить R, просто удалите R с картинки и посмотрите, что осталось:

Если вы знаете E и R и хотите определить I, удалите I и посмотрите, что осталось:

Наконец, если вы знаете I и R и хотите определить E, удалите E и посмотрите, что осталось:

В конце концов, вам придется быть знакомым с алгеброй, чтобы серьезно изучать электричество и электронику, но этот совет может облегчить запоминание ваших первых вычислений.Если вы знакомы с алгеброй, все, что вам нужно сделать, это зафиксировать E = IR в памяти и вывести из нее две другие формулы, когда они вам понадобятся!

ОБЗОР:

  • Напряжение измеряется в вольтах , обозначается буквами «E» или «V».
  • Ток измеряется в амперах , обозначается буквой «I».
  • Сопротивление измеряется в Ом , обозначается буквой «R».
  • Закон Ома: E = IR; I = E / R; R = E / I

СВЯЗАННЫЕ РАБОЧИЕ ЛИСТЫ:

Попробуйте наш калькулятор закона Ома в разделе «Инструменты».

Рабочий лист закона

Ома — Основное электричество

Позвольте электронам сами дать вам ответы на ваши собственные «практические проблемы»!

Примечания:

По моему опыту, студентам требуется много практики с анализом цепей, чтобы стать профессионалом. С этой целью инструкторы обычно предоставляют своим ученикам множество практических задач, над которыми нужно работать, и дают ученикам ответы, с которыми они могут проверить свою работу. Хотя такой подход позволяет студентам овладеть теорией схем, он не дает им полноценного образования.

Студентам нужна не только математическая практика. Им также нужны настоящие практические схемы построения схем и использование испытательного оборудования. Итак, я предлагаю следующий альтернативный подход: ученики должны построить свои собственные «практические задачи» с реальными компонентами и попытаться математически предсказать различные значения напряжения и тока. Таким образом, математическая теория «оживает», и учащиеся получают практические навыки, которых они не приобрели бы, просто решая уравнения.

Еще одна причина для следования этому методу практики — научить студентов научному методу : процессу проверки гипотезы (в данном случае математических предсказаний) путем проведения реального эксперимента.Студенты также разовьют реальные навыки поиска и устранения неисправностей, поскольку они время от времени допускают ошибки при построении схемы.

Выделите несколько минут времени со своим классом, чтобы ознакомиться с некоторыми «правилами» построения схем, прежде чем они начнутся. Обсудите эти проблемы со своими учениками в той же сократической манере, в которой вы обычно обсуждаете вопросы рабочего листа, вместо того, чтобы просто говорить им, что они должны и не должны делать. Я никогда не перестаю удивляться тому, насколько плохо студенты понимают инструкции, представленные в типичном формате лекции (монолог инструктора)!

Примечание для тех инструкторов, которые могут жаловаться на «потраченное впустую» время, необходимое ученикам для построения реальных схем вместо того, чтобы просто математически анализировать теоретические схемы:

Какова цель студентов, посещающих ваш курс?

Если ваши ученики будут работать с реальными схемами, им следует по возможности учиться на реальных схемах.Если ваша цель — обучить физиков-теоретиков, то во что бы то ни стало придерживайтесь абстрактного анализа! Но большинство из нас планируют, чтобы наши ученики что-то делали в реальном мире с образованием, которое мы им даем. «Потраченное впустую» время, потраченное на создание реальных схем, принесет огромные дивиденды, когда им придет время применить свои знания для решения практических задач.

Кроме того, когда студенты создают свои собственные практические задачи, они учатся выполнять первичных исследования , тем самым давая им возможность продолжить свое образование в области электрики / электроники в автономном режиме.

В большинстве наук реалистичные эксперименты намного сложнее и дороже, чем электрические схемы. Профессора ядерной физики, биологии, геологии и химии хотели бы, чтобы их студенты применяли передовую математику в реальных экспериментах, не представляющих опасности для безопасности и стоивших меньше, чем учебник. Они не могут, но вы можете. Воспользуйтесь удобством, присущим вашей науке, и заставьте своих учеников практиковать математику на множестве реальных схем!

Закон Ома и соотношение V-I-R

В физике есть определенные формулы, которые настолько мощны и распространены, что достигают уровня общеизвестных знаний.Студент, изучающий физику, записывал такие формулы столько раз, что запоминал их, даже не пытаясь. Безусловно, для профессионалов в этой области такие формулы настолько важны, что остаются в их сознании. В области современной физики E = m • c 2 . В области ньютоновской механики существует F net = m • a. В области волновой механики v = f • λ. А в области текущего электричества ΔV = I • R.

Преобладающим уравнением, которое пронизывает изучение электрических цепей, является уравнение

ΔV = I • R

Другими словами, разность электрических потенциалов между двумя точками в цепи ( ΔV ) эквивалентна произведению тока между этими двумя точками ( I ) и общего сопротивления всех электрических устройств, присутствующих между этими двумя точками ( R ).В остальной части этого раздела Физического класса это уравнение станет самым распространенным уравнением, которое мы видим. Это уравнение, часто называемое уравнением закона Ома , является мощным предсказателем взаимосвязи между разностью потенциалов, током и сопротивлением.

Закон Ома как предсказатель тока

Уравнение закона Ома можно переформулировать и выразить как

В качестве уравнения это служит алгебраическим рецептом для вычисления тока, если известны разность электрических потенциалов и сопротивление.Тем не менее, хотя это уравнение служит мощным рецептом решения проблем, это гораздо больше. Это уравнение указывает две переменные, которые могут повлиять на величину тока в цепи. Ток в цепи прямо пропорционален разности электрических потенциалов, приложенной к ее концам, и обратно пропорционален общему сопротивлению внешней цепи. Чем больше напряжение аккумулятора (то есть разность электрических потенциалов), тем больше ток. И чем больше сопротивление, тем меньше ток.Заряд идет с наибольшей скоростью, когда напряжение батареи увеличивается, а сопротивление уменьшается. Фактически, двукратное увеличение напряжения батареи привело бы к двукратному увеличению тока (если все остальные факторы остаются равными). А увеличение сопротивления нагрузки в два раза приведет к уменьшению тока в два раза до половины его первоначального значения.

Приведенная ниже таблица иллюстрирует это соотношение как качественно, так и количественно для нескольких цепей с различными напряжениями и сопротивлением батарей.


Строки 1, 2 и 3 показывают, что удвоение и утроение напряжения батареи приводит к удвоению и утроению тока в цепи. Сравнение строк 1 и 4 или строк 2 и 5 показывает, что удвоение общего сопротивления служит для уменьшения вдвое тока в цепи.

Поскольку на ток в цепи влияет сопротивление, в цепях электроприборов часто используются резисторы, чтобы влиять на величину тока, присутствующего в ее различных компонентах.Увеличивая или уменьшая величину сопротивления в конкретной ветви схемы, производитель может увеличивать или уменьшать величину тока в этой ветви . Кухонные приборы, такие как электрические миксеры и переключатели света, работают, изменяя ток в нагрузке, увеличивая или уменьшая сопротивление цепи. Нажатие различных кнопок на электрическом микшере может изменить режим с микширования на взбивание, уменьшив сопротивление и позволив большему току присутствовать в миксере.Точно так же поворот ручки регулятора яркости может увеличить сопротивление его встроенного резистора и, таким образом, уменьшить ток.

На схеме ниже изображена пара цепей, содержащих источник напряжения (аккумуляторная батарея), резистор (лампочка) и амперметр (для измерения тока). В какой цепи у лампочки наибольшее сопротивление? Нажмите кнопку «Посмотреть ответ», чтобы убедиться, что вы правы.


Уравнение закона Ома часто исследуется в физических лабораториях с использованием резистора, аккумуляторной батареи, амперметра и вольтметра.Амперметр — это устройство, используемое для измерения силы тока в заданном месте. Вольтметр — это устройство, оснащенное датчиками, которых можно прикоснуться к двум точкам цепи, чтобы определить разность электрических потенциалов в этих местах. Изменяя количество ячеек в аккумуляторной батарее, можно изменять разность электрических потенциалов во внешней цепи. Вольтметр может использоваться для определения этой разности потенциалов, а амперметр может использоваться для определения тока, связанного с этим ΔV.К батарейному блоку можно добавить батарею, и процесс можно повторить несколько раз, чтобы получить набор данных I-ΔV. График зависимости I от ΔV даст линию с крутизной, эквивалентной обратной величине сопротивления резистора. Это значение можно сравнить с заявленным производителем значением, чтобы определить точность лабораторных данных и справедливость уравнения закона Ома.

Величины, символы, уравнения и единицы!

Тенденция уделять внимание единицам — неотъемлемая черта любого хорошего студента-физика.Многие трудности, связанные с решением проблем, могут быть связаны с тем, что не уделили внимание подразделениям. Поскольку все больше и больше электрических величин и их соответствующих метрических единиц вводится в этот раздел учебного пособия «Физический класс», становится все более важным систематизировать информацию в своей голове. В таблице ниже перечислены некоторые из введенных на данный момент количеств. Для каждой величины также указаны символ, уравнение и соответствующие метрические единицы.Было бы разумно часто обращаться к этому списку или даже делать свою копию и добавлять к ней по мере развития модуля. Некоторые студенты считают полезным составить пятый столбец, в котором приводится определение каждой величины.

Кол-во Символ Уравнение (а) Стандартная метрическая единица Другие единицы
Разность потенциалов

(г.к.а. напряжение)

ΔV ΔV = ΔPE / Q

ΔV = I • R

Вольт (В) J / C
Текущий я I = Q / т

I = ΔV / R

Амперы (А) Усилитель или К / с

или В / Ом

Мощность п P = ΔPE / т

(еще впереди)

Ватт (Вт) Дж / с
Сопротивление р R = ρ • L / A

R = ΔV / I

Ом (Ом) В / А
Энергия E или ΔPE ΔPE = ΔV • Q

ΔPE = P • t

Джоуль (Дж) V • C или

Вт • с

(Обратите внимание, что символ C представляет собой кулоны.)

В следующем разделе Урока 3 мы еще раз рассмотрим количественную мощность. Новое уравнение мощности будет введено путем объединения двух (или более) уравнений в приведенной выше таблице.

Мы хотели бы предложить … Зачем просто читать об этом и когда можно с этим взаимодействовать? Взаимодействовать — это именно то, что вы делаете, когда используете одно из интерактивных материалов The Physics Classroom.Мы хотели бы предложить вам совместить чтение этой страницы с использованием нашего интерактивного средства построения цепей постоянного тока. Вы можете найти его в разделе Physics Interactives на нашем сайте. Построитель цепей постоянного тока предоставляет учащемуся набор для построения виртуальных цепей. Легко перетащите источник напряжения, резисторы и провода на рабочее место. Соедините их, и у вас будет схема. Добавьте амперметр для измерения тока и используйте датчики напряжения для определения падения напряжения. Это так просто. И не нужно беспокоиться о поражении электрическим током (если, конечно, вы не читаете это в ванной).


Проверьте свое понимание

1. Что из перечисленного ниже приведет к уменьшению тока в электрической цепи? Выберите все, что подходит.

а. уменьшить напряжение

г. уменьшить сопротивление

г. увеличить напряжение

г.увеличить сопротивление

2. Определенная электрическая цепь содержит батарею из трех элементов, провода и лампочку. Что из перечисленного может привести к тому, что лампа будет светить менее ярко? Выберите все, что подходит.

а. увеличить напряжение аккумулятора (добавить еще одну ячейку)

г. уменьшить напряжение аккумулятора (удалить элемент)

г.уменьшить сопротивление цепи

г. увеличить сопротивление цепи

3. Вероятно, вас предупредили, чтобы вы не прикасались к электроприборам или даже к электрическим розеткам, когда ваши руки мокрые. Такой контакт более опасен, когда ваши руки мокрые (а не сухие), потому что мокрые руки вызывают ____.

а.напряжение цепи должно быть выше

г. напряжение цепи должно быть ниже

г. ваше сопротивление будет выше

г. ваше сопротивление должно быть ниже

e. ток через тебя будет ниже

4. Если бы сопротивление цепи было утроено, то ток в цепи был бы ____.

а. треть от

г. втрое больше

г. без изменений

г. … бред какой то! Сделать такой прогноз невозможно.

5. Если напряжение в цепи увеличить в четыре раза, то ток в цепи будет ____.

а.четверть от

г. в четыре раза больше

г. без изменений

г. … бред какой то! Сделать такой прогноз невозможно.

6. Схема соединена с блоком питания, резистором и амперметром (для измерения тока). Амперметр показывает значение тока 24 мА (миллиАмпер). Определите новый ток, если напряжение источника питания было…

а. … увеличилось в 2 раза, а сопротивление осталось постоянным.

г. … увеличилось в 3 раза, а сопротивление осталось постоянным.

г. … уменьшилось в 2 раза, а сопротивление осталось постоянным.

г. … оставалось постоянным, а сопротивление увеличивалось в 2 раза.

e. … оставалось постоянным, а сопротивление увеличивалось в 4.

ф…. оставалось постоянным, а сопротивление уменьшалось в 2 раза.

г. … увеличилось в 2 раза, а сопротивление увеличилось в 2 раза.

ч. … увеличилось в 3 раза, а сопротивление уменьшилось в 2 раза.

и. … уменьшилось в 2 раза, а сопротивление увеличилось в 2 раза.

7.Используйте уравнение закона Ома, чтобы дать числовые ответы на следующие вопросы:

а. Электрическое устройство с сопротивлением 3,0 Ом позволит протекать через него току 4,0 А, если на устройстве наблюдается падение напряжения ________ Вольт.

г. Когда на электрический нагреватель подается напряжение 120 В, через нагреватель будет протекать ток 10,0 А, если сопротивление составляет ________ Ом.

г. Фонарик с питанием от 3 вольт и лампочкой с сопротивлением 60 Ом будет иметь ток ________ ампер.

8. Используйте уравнение закона Ома для определения недостающих значений в следующих схемах.

9. См. Вопрос 8 выше. В схемах схем A и B какой метод использовался для контроля тока в схемах? А в схемах схем C и D какой метод использовался для контроля тока в схемах?

Зависимость тока от напряжения — разница и сравнение

Взаимосвязь между напряжением и током

Ток и напряжение — две фундаментальные величины в электричестве.Напряжение — это причина, а ток — это следствие.

Напряжение между двумя точками равно разности электрических потенциалов между этими точками. На самом деле это электродвижущая сила (ЭДС), ответственная за движение электронов (электрический ток) по цепи. Поток электронов, приводимый в движение напряжением, называется током. Напряжение представляет собой потенциал каждого кулоновского электрического заряда для выполнения работы.

В следующем видео объясняется взаимосвязь между напряжением и током:

Схема

Электрическая цепь с источником напряжения (эл.грамм. аккумулятор) и резистор.

Источник напряжения имеет две точки с разностью электрических потенциалов. Когда между этими двумя точками существует замкнутый контур, он называется цепью, и ток может течь. При отсутствии цепи ток не будет течь, даже если есть напряжение.

Обозначения и единицы

Заглавная курсивная буква I обозначает ток. Стандартная единица измерения — Ампер (или Ампер), обозначаемая буквой A. Единица измерения тока в системе СИ — кулонов в секунду .

1 ампер = 1 кулон в секунду.

Один ампер тока соответствует одному кулону электрического заряда (6,24 x 10 18 носителей заряда), проходящего мимо определенной точки в цепи за одну секунду. Устройство, используемое для измерения тока, называется Амперметр .

Заглавная курсивная буква В обозначает напряжение.

1 вольт = 1 джоуль / кулон.

Один вольт перемещает один кулон (6,24 x 10 18 ) носителей заряда, таких как электроны, через сопротивление в 1 Ом за одну секунду.Вольтметр используется для измерения напряжения.

Поля и интенсивность

Электрический ток всегда создает магнитное поле. Чем сильнее ток, тем сильнее магнитное поле.

Напряжение создает электростатическое поле. По мере увеличения напряжения между двумя точками электростатическое поле становится более интенсивным. По мере увеличения расстояния между двумя точками, имеющими заданное напряжение по отношению друг к другу, интенсивность электростатического заряда между точками уменьшается.

Последовательные и параллельные соединения

В последовательной цепи

Напряжения суммируются для компонентов, соединенных последовательно. Токи одинаковы во всех компонентах, соединенных последовательно.

Электрические компоненты в последовательном соединении

Например, если батарея 2 В и батарея 6 В подключены последовательно к резистору и светодиоду, ток через все компоненты будет одинаковым (скажем, 15 мА), но напряжения будут разными (5 В на резисторе и 3 В на светодиод).Эти напряжения складываются с напряжением батареи: 2 В + 6 В = 5 В + 3 В.

В параллельной цепи

Сумма токов для компонентов, соединенных параллельно. Напряжения одинаковы на всех компонентах, подключенных параллельно.

Электрические компоненты при параллельном подключении

Например, если одни и те же батареи подключены к резистору и светодиоду параллельно, напряжение через компоненты будет одинаковым (8 В). Однако ток 40 мА через аккумулятор распределяется по двум путям в цепи и прерывается до 15 мА и 25 мА.

Список литературы

Закон

Ома: сопротивление и простые схемы

Цели обучения

К концу этого раздела вы сможете:

  • Объясните происхождение закона Ома.
  • Рассчитайте напряжения, токи или сопротивления по закону Ома.
  • Объясните, что такое омический материал.
  • Опишите простую схему.

Что движет током? Мы можем думать о различных устройствах, таких как батареи, генераторы, розетки и т. Д., Которые необходимы для поддержания тока.Все такие устройства создают разность потенциалов и условно называются источниками напряжения. Когда источник напряжения подключен к проводнику, он прикладывает разность потенциалов В, , которая создает электрическое поле. Электрическое поле, в свою очередь, воздействует на заряды, вызывая ток.

Ток, протекающий через большинство веществ, прямо пропорционален приложенному к нему напряжению В, . Немецкий физик Георг Симон Ом (1787–1854) первым экспериментально продемонстрировал, что ток в металлической проволоке прямо пропорционален приложенному напряжению :

.

[латекс] I \ propto {V} \\ [/ латекс].

Это важное соотношение известно как закон Ома . Его можно рассматривать как причинно-следственную связь, в которой напряжение является причиной, а ток — следствием. Это эмпирический закон, подобный закону трения — явление, наблюдаемое экспериментально. Такая линейная зависимость возникает не всегда.

Сопротивление и простые схемы

Если напряжение управляет током, что ему мешает? Электрическое свойство, препятствующее току (примерно такое же, как трение и сопротивление воздуха), называется сопротивлением R .Столкновения движущихся зарядов с атомами и молекулами вещества передают энергию веществу и ограничивают ток. Сопротивление обратно пропорционально току, или

.

[латекс] I \ propto \ frac {1} {R} \\ [/ latex].

Таким образом, например, ток уменьшается вдвое, если сопротивление увеличивается вдвое. Комбинируя отношения тока к напряжению и тока к сопротивлению, получаем

[латекс] I = \ frac {V} {R} \\ [/ латекс].

Это соотношение также называется законом Ома.Закон Ома в такой форме действительно определяет сопротивление определенных материалов. Закон Ома (как и закон Гука) не универсален. Многие вещества, для которых действует закон Ома, называются омическими . К ним относятся хорошие проводники, такие как медь и алюминий, и некоторые плохие проводники при определенных обстоятельствах. Омические материалы имеют сопротивление R , которое не зависит от напряжения В и тока I . Объект, который имеет простое сопротивление, называется резистором , даже если его сопротивление невелико.Единицей измерения сопротивления является Ом и обозначается символом Ω (заглавная греческая омега). Перестановка I = V / R дает R = V / I , и поэтому единицы сопротивления равны 1 Ом = 1 вольт на ампер:

[латекс] 1 \ Omega = 1 \ frac {V} {A} \\ [/ latex].

На рисунке 1 показана схема простой схемы. Простая схема имеет один источник напряжения и один резистор. Можно предположить, что провода, соединяющие источник напряжения с резистором, имеют незначительное сопротивление, или их сопротивление можно включить в R .

Рис. 1. Простая электрическая цепь, в которой замкнутый путь прохождения тока обеспечивается проводниками (обычно металлическими), соединяющими нагрузку с выводами батареи, представленной красными параллельными линиями. Зигзагообразный символ представляет собой единственный резистор и включает любое сопротивление в соединениях с источником напряжения.

Пример 1. Расчет сопротивления: автомобильная фара

Какое сопротивление проходит у автомобильной фары? 2.50 А течет при подаче на него 12,0 В?

Стратегия

Мы можем изменить закон Ома как указано I = V / R и использовать его, чтобы найти сопротивление.

Решение

Перестановка I = V / R и замена известных значений дает

[латекс] R = \ frac {V} {I} = \ frac {\ text {12} \ text {.} \ Text {0 V}} {2 \ text {.} \ Text {50 A}} = \ text {4} \ text {.} \ text {80 \ Omega} \\ [/ latex].

Обсуждение

Это относительно небольшое сопротивление, но оно больше, чем хладостойкость фары.Как мы увидим в разделе «Сопротивление и удельное сопротивление», сопротивление обычно увеличивается с температурой, поэтому лампа имеет меньшее сопротивление при первом включении и потребляет значительно больший ток во время короткого периода прогрева.

Сопротивление может быть разным. Некоторые керамические изоляторы, например те, которые используются для поддержки линий электропередач, имеют сопротивление 10 12 Ом или более. Сопротивление сухого человека может составлять 10 5 Ом, в то время как сопротивление человеческого сердца составляет примерно 10 3 Ом.Кусок медного провода большого диаметра длиной в метр может иметь сопротивление 10 −5 Ом, а сверхпроводники вообще не имеют сопротивления (они неомичны). Сопротивление связано с формой объекта и материалом, из которого он состоит, как будет показано в разделах «Сопротивление и удельное сопротивление». Дополнительное понимание можно получить, решив I = V / R для V , что дает

В = ИК

Это выражение для В, можно интерпретировать как падение напряжения на резисторе, вызванное протеканием тока I .Для этого напряжения часто используется фраза IR drop . Например, фара в примере 1 выше имеет падение IR на 12,0 В. Если напряжение измеряется в различных точках цепи, будет видно, что оно увеличивается на источнике напряжения и уменьшается на резисторе. Напряжение аналогично давлению жидкости. Источник напряжения подобен насосу, создающему перепад давления, вызывающему ток — поток заряда. Резистор похож на трубу, которая снижает давление и ограничивает поток из-за своего сопротивления.Здесь сохранение энергии имеет важные последствия. Источник напряжения подает энергию (вызывая электрическое поле и ток), а резистор преобразует ее в другую форму (например, тепловую энергию). В простой схеме (с одним простым резистором) напряжение, подаваемое источником, равно падению напряжения на резисторе, так как PE = q Δ V , и то же самое q протекает через каждую. Таким образом, энергия, подаваемая источником напряжения, и энергия, преобразуемая резистором, равны.(См. Рисунок 2.)

Рис. 2. Падение напряжения на резисторе в простой цепи равно выходному напряжению батареи.

Установление соединений: сохранение энергии

В простой электрической цепи единственный резистор преобразует энергию, поступающую от источника, в другую форму. Здесь о сохранении энергии свидетельствует тот факт, что вся энергия, подаваемая источником, преобразуется в другую форму только с помощью резистора. Мы обнаружим, что сохранение энергии имеет и другие важные применения в схемах и является мощным инструментом анализа схем.

Исследования PhET: закон Ома

Посмотрите, как уравнение закона Ома соотносится с простой схемой. Отрегулируйте напряжение и сопротивление и посмотрите, как изменяется ток по закону Ома. Размеры символов в уравнении изменяются в соответствии с принципиальной схемой.

Щелкните, чтобы запустить моделирование.

Сводка раздела

  • Простая схема — это схема , в которой есть один источник напряжения и одно сопротивление.
  • Одно из утверждений закона Ома дает соотношение между током I , напряжением В и сопротивлением R в простой схеме как [латекс] I = \ frac {V} {R} \\ [/ latex] .
  • Сопротивление выражается в единицах Ом (Ом), относящихся к вольтам и амперам на 1 Ом = 1 В / А.
  • На резисторе наблюдается падение напряжения IR , вызванное протекающим через него током, равным В = IR .

Концептуальные вопросы

  1. Падение напряжения IR на резисторе означает изменение потенциала или напряжения на резисторе.Изменится ли ток при прохождении через резистор? Объяснять.
  2. Каким образом падение давления в резисторе IR похоже на падение давления в жидкости, протекающей по трубе?

Задачи и упражнения

1. Какой ток протекает через лампочку фонаря на 3,00 В, когда ее горячее сопротивление составляет 3,60 Ом?

2. Вычислите эффективное сопротивление карманного калькулятора с батареей на 1,35 В, через которую протекает ток 0,200 мА.

3.Каково эффективное сопротивление стартера автомобиля, когда через него проходит 150 А, когда автомобильный аккумулятор подает на двигатель 11,0 В?

4. Сколько вольт подается для работы светового индикатора DVD-плеера с сопротивлением 140 Ом, если через него проходит 25,0 мА?

5. (a) Найдите падение напряжения в удлинителе с сопротивлением 0,0600 Ом, через который проходит ток 5,00 А. (b) Более дешевый шнур использует более тонкую проволоку и имеет сопротивление 0.300 Ом. Какое в нем падение напряжения при протекании 5.00 А? (c) Почему напряжение на любом используемом приборе снижается на эту величину? Как это повлияет на прибор?

6. ЛЭП подвешена к металлическим опорам со стеклянными изоляторами, имеющими сопротивление 1,00 × 10 9 Ом. Какой ток протекает через изолятор при напряжении 200 кВ? (Некоторые линии высокого напряжения — постоянного тока.)

Глоссарий

Закон Ома:
эмпирическое соотношение, указывающее, что ток I, пропорционален разности потенциалов В, , В, ; его часто записывают как I = V / R , где R — сопротивление
сопротивление:
электрическое свойство, препятствующее току; для омических материалов это отношение напряжения к току, R = V / I
Ом:
единица сопротивления, равная 1Ω = 1 В / А
омическое:
вид материала, для которого действует закон Ома
простая схема:
схема с одним источником напряжения и одним резистором

Избранные решения проблем и упражнения

1.0,833 А

3. 7,33 × 10 −2 Ом

5. (а) 0,300 В

(б) 1,50 В

(c) Напряжение, подаваемое на любой используемый прибор, снижается, поскольку общее падение напряжения от стены до конечного выхода прибора является фиксированным. Таким образом, если падение напряжения на удлинителе велико, падение напряжения на приборе значительно уменьшается, поэтому выходная мощность прибора может быть значительно уменьшена, что снижает способность прибора работать должным образом.

Закон

Ома: определение и взаимосвязь между напряжением, током и сопротивлением — Видео и стенограмма урока

Закон Ома

Взаимосвязь между напряжением, током и сопротивлением описывается законом Ома . Это уравнение, i = v / r , говорит нам, что ток, i , протекающий по цепи, прямо пропорционален напряжению, v , и обратно пропорционален сопротивлению, r .Другими словами, если мы увеличим напряжение, то увеличится и ток. Но, если мы увеличим сопротивление, то ток уменьшится. Мы увидели эти концепции в действии с садовым шлангом. Увеличение давления привело к увеличению потока, но изгиб шланга увеличил сопротивление, что привело к уменьшению потока.

Эта диаграмма — простой способ решать уравнения.

Как написано здесь уравнение, было бы легко использовать закон Ома, чтобы вычислить ток, если бы мы знали напряжение и сопротивление.Но что, если бы мы вместо этого захотели вычислить напряжение или сопротивление? Один из способов сделать это — переставить члены уравнения для решения других параметров, но есть более простой способ. Приведенная выше диаграмма даст нам соответствующее уравнение для решения любого неизвестного параметра без использования алгебры. Чтобы использовать эту диаграмму, мы просто закрываем параметр, который пытаемся найти, чтобы получить правильное уравнение. Это станет более понятным, когда мы начнем его использовать, поэтому давайте рассмотрим несколько примеров.

Закон Ома в действии

Ниже представлена ​​простая электрическая схема, которую мы будем использовать для выполнения наших примеров. Наш источник напряжения — это аккумулятор, подключенный к лампочке, которая обеспечивает сопротивление электрическому току. Для начала предположим, что наша батарея имеет напряжение 10 вольт, электрическая лампочка имеет сопротивление 20 Ом, и нам нужно вычислить ток, протекающий по цепи. Используя нашу диаграмму, мы закрываем параметр, который мы пытаемся найти, то есть ток, или i , и это оставляет нам напряжение v над сопротивлением r .Другими словами, чтобы найти ток, нам нужно разделить напряжение на сопротивление. Делая математические вычисления, 10 вольт, разделенные на 20 Ом, дают половину ампера тока, протекающего в цепи.

Чтобы найти ток, разделите напряжение (20 вольт) на сопротивление (20 Ом).

Теперь давайте увеличим напряжение, чтобы посмотреть, что происходит с током. Мы будем использовать ту же лампочку, но перейдем на 20-вольтовую батарею.Используя то же уравнение, что и раньше, мы разделим 20 вольт на 20 Ом, и мы получим 1 ампер тока. Как мы видим, удвоение напряжения привело к удвоению тока. Это имеет смысл, когда мы думаем о садовом шланге. Если бы мы увеличили давление в шланге, можно было бы ожидать, что поток воды также увеличится. Всегда полезно перепроверить свою работу, спросив, соответствуют ли результаты тому, что вы ожидали.

Если бы мы увеличили сопротивление лампочки, что бы вы ожидали, что произойдет с током? Чтобы выяснить это, давайте поменяем существующую лампочку на другую с сопротивлением 40 Ом.Поскольку мы все еще ищем ток, мы используем то же уравнение, что и раньше. Разделив 20 вольт на 40 Ом, мы получим половину ампера тока. Этот результат говорит нам, что удвоение сопротивления уменьшило ток вдвое. Вы этого ожидали? Если вспомнить наш шланг, логично предположить, что перегиб в шланге уменьшит поток воды, точно так же, как увеличение сопротивления в цепи уменьшит ток.

До сих пор мы только рассчитали ток в цепи, но что, если бы кто-то поменял нашу лампочку, когда мы не смотрели, и нам нужно было вычислить сопротивление новой? Что ж, мы знаем, что напряжение нашей батареи составляет 20 вольт, и мы можем измерить ток в цепи с помощью инструмента, называемого амперметром, поэтому все, что осталось, — это выполнить некоторые вычисления.Используя нашу диаграмму, мы закрываем параметр, который мы пытаемся найти, а именно сопротивление, r . Схема теперь показывает нам, что нам нужно разделить напряжение на ток. Если наш амперметр измерил ток в 5 ампер, протекающий по цепи, то сопротивление будет равно 20 вольт, разделенным на 5 ампер, что составляет 4 Ом

Чтобы определить напряжение, умножьте силу тока (3 ампера) на сопротивление (4 Ом).

Наконец, представьте, что кто-то заменил нашу батарею, и нам нужно выяснить ее напряжение.Процесс почти такой же. Мы знаем, что наша новая лампочка имеет сопротивление 4 Ом, и мы можем измерить ток в цепи с помощью амперметра. Используя диаграмму, мы покрываем напряжение v , которое говорит нам, что нам нужно умножить ток на сопротивление. Если бы амперметр измерял ток в 3 ампера, тогда напряжение было бы 3 ампера, умноженным на 4 Ом, что составляет 12 вольт. Это все, что нужно сделать. Зная любые два из трех параметров, мы всегда можем вычислить третий, используя закон Ома.

Резюме урока

Закон Ома определяет соотношение между напряжением, током и сопротивлением в электрической цепи: i = v / r . Ток прямо пропорционален напряжению и обратно пропорционален сопротивлению. Это означает, что увеличение напряжения приведет к увеличению тока, а увеличение сопротивления приведет к уменьшению тока. Зная любые два из трех параметров, мы можем вычислить третий, неизвестный параметр.Мы можем сделать это, переставив члены в уравнении закона Ома или используя диаграмму, приведенную выше в уроке.

Добавить комментарий

Ваш адрес email не будет опубликован. Обязательные поля помечены *